If the positive integer n is greater than 6, what is

This topic has expert replies
Moderator
Posts: 2058
Joined: Sun Oct 29, 2017 4:24 am
Thanked: 1 times
Followed by:5 members
If the positive integer n is greater than 6, what is the remainder when n is divided by 6?

(1) When n is divided by 9, the remainder is 2.
(2) When n is divided by 4, the remainder is 1.

The OA is the option C.

Are sufficient both statements? I need some help here. Experts, can you show me how to solve this DS question? Thanks in advanced.

User avatar
GMAT Instructor
Posts: 15539
Joined: Tue May 25, 2010 12:04 pm
Location: New York, NY
Thanked: 13060 times
Followed by:1906 members
GMAT Score:790

by GMATGuruNY » Thu Mar 22, 2018 4:47 am
-A quick lesson on remainders:
When x is divided by 5, the remainder is 3.
In other words, x is 3 more than a multiple of 5:
x = 5a + 3.

When x is divided by 7, the remainder is 4.
In other words, x is 4 more than a multiple of 7:
x = 7b + 4.

Combined, the statements above imply that when x is divided by 35 -- the LOWEST COMMON MULTIPLE OF 5 AND 7 -- there will be a constant remainder R.
Put another way, x is R more than a multiple of 35:
x = 35c + R.

To determine the value of R:
Make a list of values that satisfy the first statement:
When x is divided by 5, the remainder is 3.
x = 5a + 3 = 3, 8, 13, 18...
Make a list of values that satisfy the second statement:
When x is divided by 7, the remainder is 4.
x = 7b + 4 = 4, 11, 18...
The value of R is the SMALLEST VALUE COMMON TO BOTH LISTS:
R = 18.

Putting it all together:
x = 35c + 18.

Another example:
When x is divided by 3, the remainder is 1.
x = 3a + 1 = 1, 4, 7, 10, 13...
When x is divided by 11, the remainder is 2.
x = 11b + 2 = 2, 13...

Thus, when x is divided by 33 -- the LCM of 3 and 11 -- the remainder will be 13 (the smallest value common to both lists).
x = 33c + 13 = 13, 46, 79...
Onto the problem at hand:
M7MBA wrote:If the positive integer n is greater than 6, what is the remainder when n is divided by 6?

(1) When n is divided by 9, the remainder is 2.
(2) When n is divided by 4, the remainder is 1.
Statement 1:
In other words, n is 2 more than a MULTIPLE OF 9:
n = 9a + 2, where a is nonnegative integer.
Options for n:
2, 11, 20, 29...
Since n must be greater than 6:
If n=11, then dividing by 6 yields a remainder of 1.
If n=20, then dividing by 6 yields a remainder of 2.
Since the remainder can be different values, INSUFFICIENT.

Statement 2:
In other words, n is 1 more than a MULTIPLE OF 4:
n = 4b + 1, where b is nonnegative integer.
Options for n:
1, 5, 9, 13, 17, 21, 25, 29.,,
Since n must be greater than 6:
If n=9, then dividing by 6 yields a remainder of 3.
If n=13, then dividing by 6 yields a remainder of 1.
Since the remainder can be different values, INSUFFICIENT.

Statements combined:
When n is divided by 36 -- the LCM of 9 and 4 -- the remainder will be 29 (the smallest value common to the two lists in red):
n = 36c + 29 = 29, 65, 101...
When the values in the blue list are divided by 6, the remainder in every case is 5.
SUFFICIENT.

The correct answer is C.
Private tutor exclusively for the GMAT and GRE, with over 20 years of experience.
Followed here and elsewhere by over 1900 test-takers.
I have worked with students based in the US, Australia, Taiwan, China, Tajikistan, Kuwait, Saudi Arabia -- a long list of countries.
My students have been admitted to HBS, CBS, Tuck, Yale, Stern, Fuqua -- a long list of top programs.

As a tutor, I don't simply teach you how I would approach problems.
I unlock the best way for YOU to solve problems.

For more information, please email me (Mitch Hunt) at [email protected].
Student Review #1
Student Review #2
Student Review #3

GMAT/MBA Expert

User avatar
GMAT Instructor
Posts: 16207
Joined: Mon Dec 08, 2008 6:26 pm
Location: Vancouver, BC
Thanked: 5254 times
Followed by:1268 members
GMAT Score:770

by Brent@GMATPrepNow » Thu Mar 22, 2018 6:15 am
M7MBA wrote:If the positive integer n is greater than 6, what is the remainder when n is divided by 6?

(1) When n is divided by 9, the remainder is 2.
(2) When n is divided by 4, the remainder is 1.
Here's another (longer!) approach (MItch's solution is much nicer, but this one will also work)

Target question: What is the remainder when n is divided by 6?

Given: Positive integer n is greater than 6

Statement 1: When n is divided by 9, the remainder is 2.
------ASIDE----------------
When it comes to remainders, we have a nice rule that says:
If N divided by D leaves remainder R, then the possible values of N are R, R+D, R+2D, R+3D,. . . etc.
For example, if k divided by 5 leaves a remainder of 1, then the possible values of k are: 1, 1+5, 1+(2)(5), 1+(3)(5), 1+(4)(5), . . . etc.
------------------------------
So, from statement 1, we can conclude that some possible values of n are: 11, 20, 29, 38, 47, 56, 65. . . (since we're told n > 6, we can rule out n = 2) .
Let's test some possible values of n:
Case a: n = 11. When we divide 11 by 6, the remainder is 5
Case b: n = 20. When we divide 20 by 6, the remainder is 2
Since we cannot answer the target question with certainty, statement 1 is NOT SUFFICIENT

Statement 2: When n is divided by 4, the remainder is 1.
from statement 2, we can conclude that some possible values of n are: 9, 13, 17, 21, 25, 29, 33, 37, 41, . . . (since we're told n > 6, we can rule out n = 1 and n = 5) .
Let's test some possible values of n:
Case a: n = 9. When we divide 9 by 6, the remainder is 3
Case b: n = 13. When we divide 13 by 6, the remainder is 1
Since we cannot answer the target question with certainty, statement 2 is NOT SUFFICIENT

Statements 1 and 2 combined
Statement 1 tells us that some possible values of n are: 11, 20, 29, 38, 47, 56, 65, . . .
Statement 2 tells us that some possible values of n are: 9, 13, 17, 21, 25, 29, 33, 37, 41, 45, 49, 53, 57, 61, 65, 69 . . .
From the above, we can see that some values of n that satisfy BOTH statements are: 29, 65, etc
We might also notice that if we keep adding 36 (the least common multiple of 9 and 4) to these possible n-values, we'll get more possible n-values such as 101, 137, 173, etc
Let's test some possible values of n:
Case a: n = 29. When we divide 29 by 6, the remainder is 5
Case b: n = 65. When we divide 65 by 6, the remainder is 5
Case c: n = 101. When we divide 101 by 6, the remainder is 5
Case d: n = 137. When we divide 137 by 6, the remainder is 5
So, it certainly looks like the remainder will always be 5.
Since we can answer the target question with certainty, the combined statements are SUFFICIENT

Answer: C

Cheers,
Brent
Brent Hanneson - Creator of GMATPrepNow.com
Image

GMAT/MBA Expert

User avatar
GMAT Instructor
Posts: 1462
Joined: Thu Apr 09, 2015 9:34 am
Location: New York, NY
Thanked: 39 times
Followed by:22 members

by Jeff@TargetTestPrep » Fri Mar 23, 2018 11:55 am
M7MBA wrote:If the positive integer n is greater than 6, what is the remainder when n is divided by 6?

(1) When n is divided by 9, the remainder is 2.
(2) When n is divided by 4, the remainder is 1.
We are given that n > 6 and we need to determine the remainder when n is divided by 6.

Statement One Alone:

When n is divided by 9, the remainder is 2.

So n can be 11, 20, 29, ...

11/6 = 1 remainder 5

20/6 = 3 remainder 2

Since there could be at least two different remainders, statement one alone is not sufficient to answer the question.

Statement Two Alone:

When n is divided by 4, the remainder is 1.

So n can be 9, 13, 17, 21, 25, 29, ...


9/6 = 1 remainder 3

13/6 = 2 remainder 1

Since there could be at least two different remainders, statement two alone is not sufficient to answer the question.


Statements One and Two Together:

From the list of values of n that have a remainder of 2 when divided by 9 and the list of value of n that have a remainder of 1 when divided by 4, we see that the first value of n can be 29. By keep adding the LCM of 9 and 4, i.e., 36, we will generate a list of values of n that satisfy both criteria:

29, 65, 101, ...

29/6 = 4 remainder 5

65/6 = 10 remainder 5

101/6 = 16 remainder 5

We see that the remainder is always 5. The two statements together are sufficient to answer the question.

Answer: C

Jeffrey Miller
Head of GMAT Instruction
[email protected]

Image

See why Target Test Prep is rated 5 out of 5 stars on BEAT the GMAT. Read our reviews